Variante der Geometrischen Reihe

Aufrufe: 459     Aktiv: 14.07.2021 um 13:41

1
Hey, wie kann ich das am Besten zeigen?



Ich stehe ein wenig auf dem Schlauch. Ich weiß das die Reihe mit der Geometrischen Reihe verwandt ist, aber ich verstehe nicht so ganz wie man auf die rechte Seite kommt.
Diese Frage melden
gefragt

Punkte: 20

 
Kommentar schreiben
4 Antworten
2
Die Aufgabe erfordert etwas Durchhaltevermögen und sorgfältiges Rechnen. Hab mich dabei auch etliche Male verrechnet.
Anleitung:
Ausgangspunkt: A: \(\sum\limits_{n=0}^\infty x^n = \frac1{1-x}\).
1. Leite die Gleichung in A nach x ab. Notiere das Ergebnis (beide Seiten der Gleichung!).
2. Leite nochmal nach x ab. Notiere das Ergebnis. Ziehe die Summe auseinander (ausmultiplizieren) und erhalte fast die gewünschte Summe minus fast die Summe aus der ersten Ableitung.
3. Stelle nach der gewünschten Summe um.
Hinweis: unpassende Potenzen von x macht man durch Multiplikation oder Division mit bzw. durch geeignete Potenzen von x passend (geht, solange der Exponent fest, also ohne n ist).
Nochmal: Bei Rechenfehlern nicht aufgeben, sondern in Ruhe nochmal rechnen.
Also, fang mal an und berichte von den Ergebnissen.
Diese Antwort melden
geantwortet

Lehrer/Professor, Punkte: 38.93K

 

Genau das war ja auch meine Idee. Wissen Sie zufällig wann man bei Reihen Differenation und Summe vertauschen darf dass würde mich jetzt irgendwie interesieren.   ─   finn2000 13.07.2021 um 23:27

Weil darauf beruhen ja unsere Ansätze/Lösungen.   ─   finn2000 13.07.2021 um 23:27

Naja, wenn ich jetzt nach deiner Anleitung gehe sieht das so aus:

1.\sum\limits_{n=0}^\infinity nx^(n-1) = \frac1{(1-x)^2}
2. \sum\limits_{n=0}^\infinity n^2 x^(n-2) - nx^(n-2) = \frac2{(1-x)^3}

Ich weiß nicht genau wie hier die Eingabe sein muss, damit das richtig angezeigt wird
  ─   user4e3d2f 14.07.2021 um 08:02

Aber wie genau ziehe ich jetzt die summe auseinander?   ─   user4e3d2f 14.07.2021 um 08:04

Also hätten wir dann Σ n^2 x^(n-2) - Σ nx^(n-2). Und wie sieht dann Schritt 3 aus? Den hab ich noch nicht ganz verstanden   ─   user4e3d2f 14.07.2021 um 11:30

Ja das eine ist die Reihe aus der Aufgabe nur mit n-2 im Exponenten und die andere ist die Reihe die wir nach der ersten Abkeitung haben mit n-2 statt n-1 im Exponent   ─   user4e3d2f 14.07.2021 um 11:38

Das war ja mein Problem, ich weiß gerade nicht wie es weiter gehen soll. Wie soll ich das jetzt nach der gewünschten Summe umstellen?   ─   user4e3d2f 14.07.2021 um 12:06

Wenn ich umstelle erhalte ich folgendes:
\( \sum\limits_{n=0}^{\infty}{n^2 x^{n-2}} \) = \( \frac{2}{(1-x)^3} \) + \( \sum\limits_{n=0}^{\infty}{nx^{n-2}} \)
Aber wie bekomme ich daraus die gesuchte Summe?
  ─   user4e3d2f 14.07.2021 um 12:35

Ich verstehe nicht was genau mir der Hinweis sagen soll. Könntest du mir das bitte mal schrittweise zeigen wie es da jetzt weitergeht, damit ich das nachvollziehen kann?   ─   user4e3d2f 14.07.2021 um 13:29

Leider scheint diese Antwort Unstimmigkeiten zu enthalten und muss korrigiert werden. Mikn wurde bereits informiert.
1
Versuchs mal mit dem Ansatz:
Diese Antwort melden
geantwortet

Student, Punkte: 254

 

Ich glaube eleganter ist es sogar wenn man die Summe erstmal nur bis zu einem n0 laufen lässt dann das gleiche wie oben weiterverfolgt und am Ende n0 gegen unendlich laufen lässt dann ist nämlich auch wieder sicher gestellt das für q <1 die Reihe konvertiert. Ich hoffe es hilft:D   ─   finn2000 13.07.2021 um 22:21

Kommentar schreiben

0
Hast du schonmal probiert den rechten Ausdruck in die Form des Grenzwertes einer Geometrischen Reihe umzuformen. Wäre so mei  erster Gedanke
Diese Antwort melden
geantwortet

Student, Punkte: 254

 

Kommentar schreiben

-1
Vergess es ich glaube ich erinnere mich wie man es löst.
Diese Antwort melden
geantwortet

Student, Punkte: 254

 

Also ich hab auf Wikipedia eine Variante gesehen, wo sie mit x * (d/dx) rechnen, jedoch verstehe ich das nicht so ganz und der Weg gefällt mir auch nicht unbedingt.   ─   user4e3d2f 13.07.2021 um 22:18

Die müsste dann so ähnlich sein wie meine unten oder? Ansonsten stelle sie mal rein dann können wir schauen woran es hakt.   ─   finn2000 13.07.2021 um 22:22

Ja genau, so war es auch dort. Jedoch verstehe ich nicht so ganz was es mit diesem x d/x bzw. q d/q auf sich hat. Also wie man damit dann auf die rechte Seite kommt.   ─   user4e3d2f 13.07.2021 um 22:25

Ja damit ist einfach gemeint dass man in deinem Fall jetzt nach x ableitet und bei mir sollte es angeben dass man nach q ableitet   ─   finn2000 13.07.2021 um 22:26

Eigentlich hast du Recht und man müsste nochmal separat checken ob man hier die Reihe und die Ableitung vertauschen darf aber soweit ich mich erinnere gibt es hier keine Probleme. Wenn man es aber schön sauber haben will einfach mal schauen ob du einen Satz zum vertauschen von Differenation und (unendlichen)Summen findest :)   ─   finn2000 13.07.2021 um 22:28

Schreib mal ob es geholfen hat wenn net Frage ruhig weiter habe jetzt eh Semesterferien hahaha   ─   finn2000 13.07.2021 um 22:29

Kommst du auf die Lösung?   ─   finn2000 13.07.2021 um 22:42

Ich bin mir nicht ganz sicher ob ich das richtig verstanden hab. Was genau ist denn mit "nach x ableiten gemeint" ?   ─   user4e3d2f 13.07.2021 um 23:18

Also wenn ich z.b f(x) = x^2 habe dann ist nach x ableiten einfach das ganz normale ableiten aus der Schule. Das "nach" sagt dir nur an wonach wir ableiten also du kannst es eigentlich hier als ganz normale Ableitung lesen   ─   finn2000 13.07.2021 um 23:24

Und wir leiten die Reihe ab?   ─   user4e3d2f 13.07.2021 um 23:33

Wie genau sieht die Reihe dann aus nachdem man die ableitet?   ─   user4e3d2f 13.07.2021 um 23:33

Schau dir mal die Antwort von mikn an. Der hat es sehr schön ausgeführt   ─   finn2000 13.07.2021 um 23:35

Kommentar schreiben